2023-05-28

287: Unbounded Collection of Ordinal Numbers Is Not Set

<The previous article in this series | The table of contents of this series | The next article in this series>

A description/proof of that unbounded collection of ordinal numbers is not set

Topics


About: set

The table of contents of this article


Starting Context



Target Context


  • The reader will have a description and a proof of the proposition that any unbounded collection of ordinal numbers is not any set.

Orientation


There is a list of definitions discussed so far in this site.

There is a list of propositions discussed so far in this site.


Main Body


1: Description


Any unbounded collection, \(O\), of ordinal numbers, ordered by the \(\epsilon\) ordering, is not any set.


2: Proof


\(\cup O\) is the collection of all the ordinal numbers, because for any ordinal number, \(o_1\), there is an ordinal number, \(o_2 \in O, o_1 \in o_2\), so, \(o_1 \in \cup O\). By the Burali-Forti theorem, \(\cup O\) is not any set. So, \(O\) is not any set, because if \(O\) was a set, \(\cup O\) would a set by the union axiom.


References


<The previous article in this series | The table of contents of this series | The next article in this series>